Menu Close

Category: Others

I-have-a-question-I-am-unsure-how-this-is-done-because-I-have-never-learnt-it-How-do-you-determine-the-line-of-best-fit-

Question Number 5117 by FilupSmith last updated on 14/Apr/16 $$\mathrm{I}\:\mathrm{have}\:\mathrm{a}\:\mathrm{question}.\:\mathrm{I}\:\mathrm{am}\:\mathrm{unsure}\:\mathrm{how}\:\mathrm{this} \\ $$$$\mathrm{is}\:\mathrm{done}\:\mathrm{because}\:\mathrm{I}\:\mathrm{have}\:\mathrm{never}\:\mathrm{learnt}\:\mathrm{it}. \\ $$$$ \\ $$$$\mathrm{How}\:\mathrm{do}\:\mathrm{you}\:\mathrm{determine}\:\mathrm{the}\:\mathrm{line}\:\mathrm{of}\:\mathrm{best}\:\mathrm{fit}? \\ $$ Commented by Yozzii last updated on 14/Apr/16…

This-is-a-silly-request-but-I-think-this-app-should-have-but-the-other-direction-It-would-have-some-application-when-writing-some-matricies-and-to-more-formally-write-x-x-as-

Question Number 5090 by FilupSmith last updated on 11/Apr/16 $$\mathrm{This}\:\mathrm{is}\:\mathrm{a}\:\mathrm{silly}\:\mathrm{request},\:\mathrm{but}\:\mathrm{I}\:\mathrm{think}\:\mathrm{this} \\ $$$$\mathrm{app}\:\mathrm{should}\:\mathrm{have}\:\ddots\:\mathrm{but}\:\mathrm{the}\:\mathrm{other}\:\mathrm{direction}. \\ $$$$ \\ $$$$\mathrm{It}\:\mathrm{would}\:\mathrm{have}\:\mathrm{some}\:\mathrm{application}\:\mathrm{when} \\ $$$$\mathrm{writing}\:\mathrm{some}\:\mathrm{matricies}\:\mathrm{and}\:\mathrm{to}\:\mathrm{more}\:\mathrm{formally} \\ $$$$\mathrm{write}: \\ $$$${x}^{{x}^{…} } \:\:\:\:\:\:\mathrm{as}\:\:\:\:\:\:\:\:{x}^{{x}^{\ddots} }…

An-engine-is-pumping-water-from-a-well-25m-deep-It-discharges-0-4-m-3-of-water-each-second-with-a-velocity-of-12-ms-1-Find-the-power-of-the-pump-given-that-the-density-of-water-is-1000-kg-m-

Question Number 136110 by physicstutes last updated on 18/Mar/21 $$\mathrm{An}\:\mathrm{engine}\:\mathrm{is}\:\mathrm{pumping}\:\mathrm{water}\:\mathrm{from}\:\mathrm{a}\:\mathrm{well}\:\mathrm{25m}\:\mathrm{deep}.\:\mathrm{It}\:\mathrm{discharges} \\ $$$$\mathrm{0}.\mathrm{4}\:\mathrm{m}^{\mathrm{3}} \:\mathrm{of}\:\mathrm{water}\:\mathrm{each}\:\mathrm{second}\:\mathrm{with}\:\mathrm{a}\:\mathrm{velocity}\:\mathrm{of}\:\mathrm{12}\:\mathrm{ms}^{−\mathrm{1}} .\:\mathrm{Find}\:\mathrm{the}\: \\ $$$$\mathrm{power}\:\mathrm{of}\:\mathrm{the}\:\mathrm{pump}\:\mathrm{given}\:\mathrm{that}\:\mathrm{the}\:\mathrm{density}\:\mathrm{of}\:\mathrm{water}\:\mathrm{is}\:\mathrm{1000}\:\mathrm{kg}\:\mathrm{m}^{−\mathrm{3}} . \\ $$$$\mathrm{take}\:\boldsymbol{\mathrm{g}}\:=\:\mathrm{10}\:\mathrm{ms}^{−\mathrm{2}} \\ $$ Commented by mr W…

cos-1-43-3-pi-1-2-cos-1-43-3-2pi-2-2-cos-1-43-3-3pi-3-2-api-Find-a-

Question Number 136104 by Dwaipayan Shikari last updated on 18/Mar/21 $$\frac{{cos}\left(\mathrm{1}+\sqrt{\frac{\mathrm{43}}{\mathrm{3}}}\right)\pi}{\mathrm{1}^{\mathrm{2}} }+\frac{{cos}\left(\mathrm{1}+\sqrt{\frac{\mathrm{43}}{\mathrm{3}}}\right)\mathrm{2}\pi}{\mathrm{2}^{\mathrm{2}} }+\frac{{cos}\left(\mathrm{1}+\sqrt{\frac{\mathrm{43}}{\mathrm{3}}}\right)\mathrm{3}\pi}{\mathrm{3}^{\mathrm{2}} }+…={a}\pi \\ $$$${Find}\:{a} \\ $$ Terms of Service Privacy Policy Contact: info@tinkutara.com

The-sign-for-is-not-congruent-to-is-present-on-the-keyboard-But-its-opposite-is-congruent-to-doesn-t-exist-Although-by-adding-two-signs-and-we-can-make-the-required-sign-b

Question Number 5013 by Rasheed Soomro last updated on 01/Apr/16 $${The}\:{sign}\:{for}\:“\:{is}\:{not}\:{congruent}\:{to}'' \\ $$$${is}\:{present}\:{on}\:{the}\:{keyboard}\:\left(\ncong\right). \\ $$$${But}\:{its}\:{opposite}\:“\:{is}\:{congruent}\:{to}'' \\ $$$${doesn}'{t}\:{exist}. \\ $$$${Although}\:{by}\:{adding}\:{two}\:{signs}\:'='\:{and}\:'\sim' \\ $$$${we}\:{can}\:{make}\:{the}\:{required}\:{sign}\:\left(\overset{\sim} {=}\right)\:{but} \\ $$$${I}\:{think}\:{it}\:{should}\:{be}\:{directly}\:{present}. \\…

sin-2-1-3-sin-2-2-2-3-sin-3-2-3-3-pi-b-1-a-b-pi-b-Find-a-b-

Question Number 136070 by Dwaipayan Shikari last updated on 18/Mar/21 $$\frac{{sin}\left(\sqrt{\mathrm{2}}\right)}{\mathrm{1}^{\mathrm{3}} }+\frac{{sin}\left(\mathrm{2}\sqrt{\mathrm{2}}\right)}{\mathrm{2}^{\mathrm{3}} }+\frac{{sin}\left(\mathrm{3}\sqrt{\mathrm{2}}\right)}{\mathrm{3}^{\mathrm{3}} }+…=\frac{\pi^{{b}} +\mathrm{1}}{\:{a}\sqrt{{b}}}−\frac{\pi}{{b}} \\ $$$${Find}\:{a}−{b} \\ $$ Answered by mnjuly1970 last updated on…

is-possible-to-proof-that-f-x-e-cx-obey-f-x-y-f-x-f-y-using-e-x-n-0-x-n-n-

Question Number 4962 by 123456 last updated on 27/Mar/16 $$\mathrm{is}\:\mathrm{possible}\:\mathrm{to}\:\mathrm{proof}\:\mathrm{that} \\ $$$${f}\left({x}\right)={e}^{{cx}} \\ $$$$\mathrm{obey} \\ $$$${f}\left({x}+{y}\right)={f}\left({x}\right){f}\left({y}\right) \\ $$$$\mathrm{using} \\ $$$${e}^{{x}} =\underset{{n}=\mathrm{0}} {\overset{+\infty} {\sum}}\frac{{x}^{{n}} }{{n}!} \\…

x-cos-sin-y-sin-cos-z-sin-r-x-e-x-y-e-y-z-e-z-r-r-r-

Question Number 4922 by 123456 last updated on 22/Mar/16 $$\begin{cases}{{x}\left(\rho,\theta,\psi\right)=\rho\:\mathrm{cos}\:\theta+\psi\:\mathrm{sin}\:\theta}\\{{y}\left(\rho,\theta,\psi\right)=\rho\:\mathrm{sin}\:\theta+\psi\:\mathrm{cos}\:\theta}\\{{z}\left(\rho,\theta,\psi\right)=\psi\:\mathrm{sin}\:\theta}\end{cases} \\ $$$$\boldsymbol{{r}}\left(\rho,\theta,\psi\right)={x}\left(\rho,\theta,\psi\right)\:\boldsymbol{{e}}_{{x}} +{y}\left(\rho,\theta,\psi\right)\:\boldsymbol{{e}}_{{y}} +{z}\left(\rho,\theta,\psi\right)\:\boldsymbol{{e}}_{{z}} \\ $$$$\frac{\partial\boldsymbol{{r}}}{\partial\rho}=? \\ $$$$\frac{\partial\boldsymbol{{r}}}{\partial\theta}=? \\ $$$$\frac{\partial\boldsymbol{{r}}}{\partial\psi}=? \\ $$ Commented by prakash…

f-x-0-is-a-even-function-or-a-odd-function-

Question Number 4905 by 123456 last updated on 20/Mar/16 $${f}\left({x}\right)=\mathrm{0}\:\mathrm{is}\:\mathrm{a}\:\mathrm{even}\:\mathrm{function}\:\mathrm{or}\:\mathrm{a}\:\mathrm{odd}\:\mathrm{function}? \\ $$ Commented by Yozzii last updated on 20/Mar/16 $${This}\:{is}\:{indeterminate}\:{I}\:{think}\:{since}\:{f}\left({x}\right)=\mathrm{0} \\ $$$${satisfies}\:{both}\:{f}\left({x}\right)=−{f}\left({x}\right)=\mathrm{0} \\ $$$${and}\:{f}\left(−{x}\right)=\mathrm{0}={f}\left({x}\right). \\…